You are on page 1of 15

The Epsilon-Delta Limit Denition:

A Few Examples
Nick Rauh
1. Prove that lim
xa
x
2
= a
2
. (Since we leave a arbitrary, this is the
same as showing x
2
is continuous.)
Proof: Let > 0. We wish to nd > 0 such that for any x R,
0 < |x a| < implies |x
2
a
2
| < . We claim that the choice
= min
_

|2a| + 1
, 1
_
is an appropriate choice of . First note that |2a| is always nonneg-
ative, so |2a| + 1 is always positive. This means

|2a|+1
> 0 for any
a R, so we have actually chosen > 0. Next we observe that if
|x a| < = min
_

|2a|+1
, 1
_
, then
|x
2
a
2
| = |x + a||x a| (factoring)
< (|2a| + 1)|x a| (|x + a| < |2a| + 1 if |x a| < 1)
< (|2a| + 1)

|2a| + 1
_
|x a| <

|2a| + 1
_
= .
Thus, we see that this choice of forces |x
2
a
2
| < whenever
0 < |x a| < . It follows that lim
xa
x
2
= a
2
.
1
Discussion Once we have a > 0 that we think could work, all we have to do is
plug it in as we did above and verify. But how did we come up with
our particular choice of > 0? We basically had to work backwards.
We started with the idea that given > 0 we want > 0 such that
for any x R, 0 < |x a| < implies |x
2
a
2
| < . We then had two
inequalities to work with and somehow wanted to use them to come
up with an expression for .
We might rst note that we can factor |x
2
a
2
| into |x+a||xa|. We
could then rewrite our task to be nding a 0 such that |x a| <
implies |x + a||x a| < . The nice thing about this new task is that
we have a common coecient in both of our inequalities:
|x a| < |x + a| |x a| <
Perhaps this could be our in to combining the inequalities and get-
ting an expression for . Our rst step towards this would be isolating
the common coecient in our expression for :
|x a| <

|x + a|
.
We then note that if we were to choose /|x + a|, then it would
certainly be the case that
|x a| <

|x + a|
|x a||x + a| < .
However, > 0 would then depend on x. However, we want |xa| < .
This means we actually want x to depend on ! So we need to nd a
choice of > 0 such that does not depend on x.
We know that choosing /|x +a| will get the job done. The ques-
tion is how we can choose in this way. Well, we know that whenever
we have one > 0 that works, any smaller value will also work. This
means we can assume < 1. By choosing 1, we would have
|x a| < 1 whenever |x a| < .
This is the part where the reasoning gets tricky. If |x a| 1, this
is the same as saying that x is no further away from a than a dis-
tance of 1. If x is no further away from a than a distance of 1, what
2
can we say about |x + a|? Well, we can say that x + a is no further
away from 2a than a distance of 1. We can then write |x+a| < |2a|+1.
We then notice that we have just created a bound for |x + a|, which
was the term in = /|x+a| that made this choice of dependent on
x. We note that if we replace the denominator with |2a| + 1, we get
|x a| < = min
_

|2a| + 1
, 1
_
<

|x + a|
.
The reason that we have to take the minimum of /(|2a| + 1) and 1
is because we had shown that |x + a| < |2a| + 1 on the condition that
|x a| < 1. If we had chosen to bound above by some other
valuesay, < 20then we would nd that |x+a| < |2a| +20 on the
condition that |x a| < 20, and thus choose
= min
_

|2a| + 20
, 20
_
.
We have thus constructed a > 0 that we think will work. All that
is left for us to do is go back and make sure that for this > 0,
0 < |x a| < implies |x
2
a
2
| < , as we did in the proof.
3
2. Prove that lim
x2
x
3
= 8. (This proves that x
3
is continuous at
x = 2.)
Proof: Let > 0. We want > 0 such that for any x R,
0 < |x 2| < forces |x
3
8| < . We claim that
= min
_

19
, 1
_
gets the job done. To see that this, suppose |x 2| < = min
_

19
, 1
_
.
Then we must have
|x
3
8| = |x 2||x
2
+ 2x + 4| (factoring)
< |x 2|19 (|x
2
+ 2x + 4| < 19 if |x 2| < 1)
<

19
19
_
|x 2| <

19
_
= .
Thus, we have that this forces |x
3
8| < whenever 0 < |x 2| < .
It follows that lim
x2
x
3
= 8.
4
Discussion Again, we have created a > 0 as if by magic. How would we go
about nding such a ? As last time, we were given > 0 and wanted
> 0 such that for any x R, 0 < |x 2| < implies |x
3
8| < .
We again had two inequalities to work with and somehow wanted to
use them to create an expression for .
Since we are again dealing with a polynomial, our gut instinct is to
attempt to factor it. This gives |x
3
8| = |x2||x
2
+2x+4|, allowing
us to rephrase our question. We want > 0 such that |x 2| <
implies |x 2||x
2
+ 2x + 4| < . Just like last time we might then
consider
=

|x
2
+ 2x + 4|
,
since
|x 2| < =

|x
2
+ 2x + 4|
|x 2||x
2
+ 2x + 4| < .
However, this choice of is dependent on x, which is no good. We
can save ourselves the same way we did last time, though. If we have
a good choice of > 0, then any smaller choice will also work, so we
can place the restriction 1. This would then mean that whenever
|x 2| < , then |x 2| < 1. Unravelling this inequality, we have
|x 2| < 1 1 < x 2 < 1 1 < x < 3.
(Here, is the mathematical symbol for these two statements are
equivalent.) Since |x
2
+ 2x + 4| is the part of our expression that
is causing problems, we will try and use this restriction 1 < x < 3 to
restrict |x
2
+ 2x + 4|.
We begin by noting that 1 < x < 3 implies the following two inequal-
ities:
2 < 2x < 6 and 1 < x
2
< 9.
Putting these together gives us the following:
3 < x
2
+ 2x < 15 7 < x
2
+ 2x + 4 < 19.
Here comes the slightly tricky part in the reasoning. Since |x2| < 1,
x can be no further away from 2 than a distance of 1. This means that
x is positive. If x is positive, then so is x
2
+ 2x + 4, so we must have
x
2
+ 2x + 4 = |x
2
+ 2x + 4| 7 < |x
2
+ 2x + 4| < 19.
5
In this way we have bounded |x
2
+2x+4|. Going back to our previous
argument, we note that as long as we choose /|x
2
+ 2x + 4|, it
should be a good choice of . Since |x
2
+2x+4| < 19 when |x2| < 1,
we might then choose
= min
_

19
, 1
_
,
since 19 > |x
2
+ 2x + 4| makes
|x 2| <

19
<

|x
2
+ 2x + 4|
.
To formulate a proof, it would then remain for us to show that this
gives |x
3
8| < whenever |x 2| < , as we did in the proof above.
6
3. Prove that lim
x1
|x + 2| = 3. (This shows |x + 2| is continuous at
x = 1.)
Proof: Suppose we are given > 0. We then wish to nd > 0 such
that for any x R, 0 < |x 1| < implies ||x +2| 3| < . We claim
that the choice
= min{, 1}
is appropriate. To verify this, suppose |x 1| < = min{, 1}. Then
|x 1| < < x 1 < (unravelling the absolute value)
3 < x + 2 < 3 + (adding 3 to all sides)
3 < |x + 2| < 3 + (|x + 2| > 0 if |x 1| < 1)
< |x + 2| 3 < (subtracting 3 from all sides)
||x + 2| 3| < (combining the inequalities)
Thus, we see that this choice of forces ||x + 2| 3| < whenever
0 < |x 1| < . Thus, lim
x1
|x + 2| = 3.
7
Discussion Again, we found a > 0 that works, so lets see how we might have
found it. Unlike last time, we have no polynomials to factor. We do,
however, have an excessive number of absolute value symbols. When in
doubt, unravel those things. Well start (as before) with our inequality
for :
||x + 2| 3| < < |x + 2| 3 <
3 < |x + 2| < 3 + .
We now have an absolute value wedged between two values. If we
unravel this thing, we actually end up with two sets of inequalities.
The rst type of inequality we have seen before:
(3 + ) < x + 2 < 3 + 5 < x < 1 + .
The second type is something new:
x + 2 > 3 or x + 2 < (3 ).
The tricky part in the reasoning is then as follows. We are of the
mindset that is going to be arbitrarily small, so for the values of
that are worth considering, (3 ) < 0. However, if we set the
requirement 1, which is completely within our right, we would
then have |x 1| < 1, so x must be positive. If x > 0, then certainly
x + 2 > 0. Thus, in the above inequality, the option
x + 2 < (3 )
makes no sense, since it claims a positive number is less than a negative
number. We must then choose
x + 2 > 3 x > 1 .
Thus, we have the following two sets of inequalities:
5 < x < 1 + and x > 1 .
We can then try to combine them. We note that since 5 < 1
and 1 < x, we can combine them in the following way:
5 < 1 < x < 1 + 1 < x < 1 +
< x 1 < .
8
Since one of our bounds is the negation of the other, we may then fold
this back into an absolute value inequality:
|x 1| < .
It might then seem like is an appropriate choice of > 0. However,
we have to remember that part of our reasoning involved forcing 1,
so we must instead suggest
= min{, 1}.
To complete a proof, we would then check whether this choice of is
such that ||x + 2| 3| < whenever 0 < |x 1| < , as we did in our
proof above.
9
4. Prove that lim
x1
(4 + x 3x
3
) = 2. (This proves that 4 + x 3x
3
is continuous at x = 1.)
Proof: Suppose > 0. We want to nd some > 0 such that for any
x R, 0 < |x 1| < forces |(4 + x 3x
3
) 2| < . We claim that
= min
_

20
, 1
_
would be an alright choice for . To check this, suppose |x 2| < = min
_

20
, 1
_
.
Then we see that
|(4 + x 3x
3
) 2| = | 3x
3
+ x + 2| (simplication)
= |x 1|| 3x
2
+ x + 2| (factoring)
< |x 1|20 (| 3x
2
+ x + 2| < 20 if |x 1| < 1)
<

20
20
_
|x 1| <

20
_
= .
Thus, we have that this forces |(4 + x 3x
3
) 2| < whenever
0 < |x 1| < . Thus, lim
x1
(4 + x 3x
3
) = 2.
10
Discussion Once again, it is time to discuss how we might nd the that we did.
We want to nd > 0 such that |(4 + x 3x
3
) 2| < whenever
0 < |x 1| < , so we are (as always) attempting to coerce something
useful out of two inequalities. Since we are back to dealing with poly-
nomials, our gut instinct is to factor. Our polynomial is even messier
than the previous two we have looked at, but we argue that we can
still use a factoring trick. We note that
|(4 + x 3x
3
) 2| = | 3x
3
+ x + 2| = |x 1|| 3x
2
3x 2|.
Now our problem is equivalent to wanting > 0 such that |x 1| <
implies |x 1|| 3x
3
+ x + 2| < . Our natural rst guess for is
=

| 3x
2
3x 2|
,
since
|x1| < =

| 3x
2
3x 2|
|x1|| 3x
2
3x2| < .
However, just as with our two previous polynomial examples, this
is dependent on x, so we need a dierent . Again, if we have a
> 0 that works, then any smaller choice will also work, so place the
restriction 1. This gives that whenever |x1| < , then |x1| < 1.
Unravelling this inequality,
|x 1| < 1 1 < x 1 < 1 0 < x < 2.
We can see that 0 < x < 2 implies the following inequalities:
0 > 3x
2
> 12 and 0 > 3x > 6.
Putting these together gives us the following:
18 < 3x
2
3x < 0 20 < 3x
2
3x 2 < 2.
Since this inequality gives that 3x
2
3x2 is negative for |x1| < 1,
we must nd that | 3x
2
3x 2| = (3x
2
3x 2), and thus
2 < | 3x
2
3x 2| < 20.
Thus, we have bounded | 3x
2
3x 2|. Going back to our previous
argument, as long as we choose /| 3x
2
3x 2|, it should be
a good choice of . Since | 3x
2
3x 2| < 20 when |x 1| < 1, we
might then choose
= min
_

20
, 1
_
.
11
For Fun! A big question we might ask is, when considering the limit of a poly-
nomial, will we always be able to use the factoring trick? Much to
our delight, the answer is yes! To see this, consider what it means
to be able to factor out a (x c) from a polynomial. It means that
the polynomial is equal to zero when we plug c in. If we have some
polynomial a
n
x
n
+ a
n1
x
n1
+ . . . + a
1
x + a
0
, then consider the new
polynomial
(a
n
x
n
+a
n1
x
n1
+. . . +a
1
x+a
0
) (a
n
c
n
+a
n1
c
n1
+. . . +a
1
c+a
0
),
which can be rewritten
a
n
(x
n
c
n
) + a
n1
(x
n1
c
n1
) + . . . + a
1
(x c) + a
0
(1 1).
This is a polynomial that zeros out when we plug in c, so we should
be able to factor out (x c). Suppose we were considering the limit
of the original polynomial as x approaches c, then.
lim
xc
(a
n
x
n
+ a
n1
x
n1
+ . . . + a
1
x + a
0
).
We would guess that the limit is exactly what happens when we plug
c ini.e.,
lim
xc
(a
n
x
n
+a
n1
x
n1
+. . .+a
1
x+a
0
) = (a
n
c
n
+a
n1
c
n1
+. . .+a
1
c+a
0
).
When we try to do an epsilon-delta proof to show this is true, given
> 0 we would be considering
|(a
n
x
n
+a
n1
x
n1
+. . .+a
1
x+a
0
)(a
n
c
n
+a
n1
c
n1
+. . .+a
1
c+a
0
)| < ,
which can be rewritten
|a
n
(x
n
c
n
) + a
n1
(x
n1
c
n1
) + . . . + a
1
(x c) + a
0
(1 1)|.
This is precisely the polynomial we argued we could factor (xc) out
of, though! This means that whenever were considering |x c| <
for f(x) a polynomial, we can factor a |x c| out of the |f(x) L|.
(Swank.)
12
5. Prove that lim
x2
4x+1
3x4
= 9/2. (This proves that
4x+1
3x4
is continuous
at x = 2.)
Proof: Let > 0 be given. We wish to nd some > 0 such that for
any x R, 0 < |x 2| < forces |(4x + 1)/(3x 4) 9/2| < . We
claim that
= min
_
2
19
,
1
3
_
would work. To check this, suppose |x 2| < = min
_
2
19
,
1
3
_
. Sim-
plifying |(4x + 1)/(3x 4) 9/2|, we get

4x + 1
3x 4

9
2

2(4x + 1) 9(3x 4)
(3x 4)2

19(x 2)
2(3x 4)

Using our choice of , we then notice

19(x 2)
2(3x 4)

= |x 2|

19
2(3x 4)

< |x 2|
19
2
_

19
2(3x 4)

<
19
2
if |x 2| <
1
3
_
<
_
2
19
__
19
2
_ _
|x 2| <
2
19
_
= .
Thus, this forces |(4x+1)/(3x4)9/2| < whenever 0 < |x 2| < .
It follows that lim
x2
4x+1
3x4
= 9/2.
13
Discussion How would we nd this time? We have the two inequalities |x2| <
and |(4x + 1)/(3x 4) 9/2| < . Lets see if we can massage our
inequality into something that looks more helpful. We did that in our
proof above:

4x + 1
3x 4

9
2

= |x 2|

19
2(3x 4)

We note now that our inequalities each have a common coecient:


|x 2| < and |x 2|

19
2(3x 4)

< .
This common coecient is a good in for understanding how one of
our inequalities might aect the other. We note that if we were to
choose
=

2(3x 4)
19

,
then we would certainly have
|x 2| < =

2(3x 4)
19

|x 2|

19
2(3x 4)

< .
However, something we are getting well used to seeing is that this
choice of depends on x. Thus, we need another . Since any time we
choose a value for that works a smaller one will also work, we can
request 1, as we have done time and time again before. This gives
us
|x 2| < 1 1 < x 2 < 1 1 < x < 3.
We then see that 1 < x < 3 implies the following:
1 < x < 3 3 < 3x < 9 1 < 3x 4 < 5.
If we were to act as we have in the previous examples, we would then
want to say something to the eect of since 1 < 3x 4, we can
choose = |2(1)/19| < |2(3x 4)/19| . However, is this actually
true for all 1 < x < 3? Unfortunately, no. If we plug in x = 4/3,
we nd that our would have to be less than zero, which is no good.
So it is the value x = 4/3 that is screwing us over because our upper
bound for , |2(3x 4)/19| , zeros out there.
14
The question, then, is what can be done about it. Well, we declared
1 so that we would have |x 2| < 1. The problem was that we
found |4/3 2| = 2/3 < 1. We then might want to make a tighter
bound on such that |4/32| is not less than . Since |4/32| = 2/3,
we might then choose 1/3. So now we go about the steps we took
for 1, except now use 1/3.
Since 1/3 and |x 2| < , then |x 2| < 1/3. We then unravel
this absolute value inequality:
|x 2| <
1
3

1
3
< x 2 <
1
3

5
3
< x <
7
3
.
We then see that
5
3
< x <
7
3
implies the following:
5
3
< x <
7
3
5 < 3x < 7 1 < 3x 4 < 3.
Now we have that 1 < 3x4 < 3, which implies that 3x4 is positive.
This is fantastic, because we can then write 3x 4 = |3x 4|, and
thus
1 < |3x 4| < 3
Since we have that 1 < |3x 4|, we might then choose
=

2(1)
19

< |2(3x 4)/19| .


This is the same as before, it would appear. However, note that this
time we have required that 1/3 rather than the 1 of before.
Thus, the we have chosen the second time is
= min
_
2
19
,
1
3
_
instead of = min{2/19, 1}. While they are subtly dierent, we can
see that the dierence does matterone value of may still work while
we saw that the other would not! It would then remain to check that
our chosen value of gets the job done, as we did in the proof above.
15

You might also like